Hostname: page-component-7bb8b95d7b-wpx69 Total loading time: 0 Render date: 2024-09-28T19:35:29.440Z Has data issue: false hasContentIssue false

Equal-Sum-Product problem II

Published online by Cambridge University Press:  13 September 2023

Maciej Zakarczemny*
Affiliation:
Department of Applied Mathematics, Cracow University of Technology, Warszawska 24, 31-155 Kraków, Poland
Rights & Permissions [Opens in a new window]

Abstract

In this paper, we present the results related to a problem posed by Andrzej Schinzel. Does the number $N_1(n)$ of integer solutions of the equation

$$ \begin{align*}x_1+x_2+\cdots+x_n=x_1x_2\cdot\ldots\cdot x_n,\,\,x_1\ge x_2\ge\cdots\ge x_n\ge 1\end{align*} $$
tend to infinity with n? Let a be a positive integer. We give a lower bound on the number of integer solutions, $N_a(n)$, to the equation
$$ \begin{align*}x_1+x_2+\cdots+x_n=ax_1x_2\cdot\ldots\cdot x_n,\,\, x_1\ge x_2\ge\cdots\ge x_n\ge 1.\end{align*} $$
We show that if $N_2(n)=1$, then the number $2n-3$ is prime. The average behavior of $N_2(n)$ is studied. We prove that the set $\{n:N_2(n)\le k,\,n\ge 2\}$ has zero natural density.

Type
Article
Copyright
© The Author(s), 2023. Published by Cambridge University Press on behalf of Canadian Mathematical Society

1 Introduction

Let $\mathbb {N}=\{1,2,3,\ldots \}$ denote the set of all natural numbers (i.e., positive integers). Equal-Sum-Product Problem is relatively easy to formulate but still unresolved (see [Reference Guy4]). Some early research focused on estimating the number of solutions, $N_1(n)$ , to the equation

(1.1) $$ \begin{align} x_1+x_2+\cdots+x_n=x_1x_2\cdot\ldots\cdot x_n,\,\, x_1\ge x_2\ge\cdots\ge x_n\ge 1, \end{align} $$

which can be found in [Reference Ecker3, Reference Nyblom8]. Schinzel asked in papers [Reference Schinzel10, Reference Schinzel11] if the number $N_1(n)$ tends toward infinity with n. This conjecture is yet to be proven. In [Reference Zakarczemny15], it was shown that the set $\{n:N_1(n)\le k,\,n\in \mathbb {Z},\,n\ge 2\}$ has zero natural density for all natural k. It is worth noting that the classical Diophantine equation $x_1^2+x_2^2+x_3^2=3x_1x_2x_3$ was investigated by Markoff (1879), as mentioned in [Reference Cassels1, Reference Markoff7]. Additionally, Hurwitz (see [Reference Hurwitz5]) examined the family of equations $x_1^2+x_2^2+\cdots +x_n^2=ax_1x_2\cdot \ldots \cdot x_n,$ where $a,n~\in \mathbb {N},\,n\ge 3.$ Let us now assume that $a,n~\in \mathbb {N},\,n\ge 2.$ In this paper, we provide a lower bound for the number $N_a(n)$ of integer solutions $(x_1,\,x_2,\ldots ,x_n)$ of the equation

(1.2) $$ \begin{align} x_1+x_2+\cdots+x_n=ax_1x_2\cdot\ldots\cdot x_n \end{align} $$

such that $x_1\ge x_2\ge \cdots \ge x_n\ge 1$ . Some of the results presented can be generalized to the case of the equation

(1.3) $$ \begin{align} b(x_1+x_2+\cdots+x_n)=ax_1x_2\cdot\ldots\cdot x_n, \end{align} $$

where $a,b$ are positive integers. In the case $a=1,\,b=n$ , the equation

$$ \begin{align*}n(x_1+\cdots+x_n)=x_1\cdot x_2\cdot\ldots\cdot x_n\end{align*} $$

is called Erdós last equation (see [Reference Guy4, Reference Shiu12, Reference Takeda13]). Equation (1.3) is related to the problem of finding numbers divisible by the sum and product of their digits. It is worth noting that if equation (1.2) has solutions, then $a\le n$ .

2 Basic results

In this section, we discuss the necessary basic results. First, we will show that the number of solutions $N_a(n)$ is finite for any fixed a and n.

Lemma 2.1 Let n be a natural number. If $x_1,x_2,\ldots ,x_n$ are any real numbers, then the following formula holds:

(2.1) $$ \begin{align} \nonumber \left(a\prod\limits_{i=1}^{n-1} x_i-1\right)\left(ax_{n}-1\right)+a\sum\limits_{s=1}^{n-2}\left(\left(\prod\limits_{i=1}^s x_i-1\right)\left(x_{s+1}-1\right)\right)= \\ a^2\prod\limits_{i=1}^n x_i-a\sum\limits_{i=1}^nx_i+a(n-2)+1. \end{align} $$

Proof Let us denote equation (2.1) as $T(n)$ . We want to show by induction that $T(n)$ holds for every natural number n. The cases $n = 1$ and $n=2$ are trivial: $(a-1)(ax_1-1)=a^2x_1-ax_1-a+1,\,(ax_1-1)(ax_2-1)=a^2x_1x_2-a(x_1+x_2)+1.$ In both cases, equality is true. Therefore, the base step of the induction is satisfied, as $T(1)$ and $T(2)$ hold. Let us assume now that $n \ge 3$ and $T(n-1)$ holds, i.e., the following equality is true:

(2.2) $$ \begin{align} \nonumber \left(a\prod\limits_{i=1}^{n-2} x_i-1\right)\left(ax_{n-1}-1\right)+a\sum\limits_{s=1}^{n-3}\left(\left(\prod\limits_{i=1}^s x_i-1\right)\left(x_{s+1}-1\right)\right)=\\ a^2\prod\limits_{i=1}^{n-1} x_i-a\sum\limits_{i=1}^{n-1}x_i+a(n-3)+1. \end{align} $$

In the inductive step, we will be using the equivalent form of equation (2.2):

(2.3) $$ \begin{align} \nonumber a\sum\limits_{s=1}^{n-3}\left(\left(\prod\limits_{i=1}^s x_i-1\right)\left(x_{s+1}-1\right)\right)=\\ -\left(a\prod\limits_{i=1}^{n-2} x_i-1\right)\left(ax_{n-1}-1\right)+a^2\prod\limits_{i=1}^{n-1} x_i-a\sum\limits_{i=1}^{n-1}x_i+a(n-3)+1. \end{align} $$

To prove the inductive step, i.e., to show that $T(n-1)$ implies $T(n)$ for $n \ge 3$ , we will use the following algebraic identities that can be verified directly:

(2.4) $$ \begin{align} \left(a\prod\limits_{i=1}^{n-1} x_i-1\right)\left(ax_{n}-1\right)=a^2\prod\limits_{i=1}^n x_i-ax_n+1-a\prod\limits_{i=1}^{n-1} x_i, \end{align} $$
(2.5) $$ \begin{align} \nonumber a\sum\limits_{s=1}^{n-2}\left(\left(\prod\limits_{i=1}^s x_i-1\right)\left(x_{s+1}-1\right)\right)=\\ a\left(\prod\limits_{i=1}^{n-2} x_i-1\right)\left(x_{n-1}-1\right)+ a\sum\limits_{s=1}^{n-3}\left(\left(\prod\limits_{i=1}^s x_i-1\right)\left(x_{s+1}-1\right)\right). \end{align} $$

Let us proceed to the proof of the inductive step. We want to show $T(n)$ assuming $T(n-\text {1})$ . Let us start by transforming the left side of $T(n)$ using equations (2.4) and (2.5)

(2.6) $$ \begin{align} \nonumber \left(a\prod\limits_{i=1}^{n-1} x_i-1\right)\left(ax_{n}-1\right)+a\sum\limits_{s=1}^{n-2}\left(\left(\prod\limits_{i=1}^s x_i-1\right)\left(x_{s+1}-1\right)\right)=\\ \nonumber a^2\prod\limits_{i=1}^n x_i-ax_n+1-a\prod\limits_{i=1}^{n-1} x_i+\\ +a\left(\prod\limits_{i=1}^{n-2} x_i-1\right)\left(x_{n-1}-1\right)+ a\sum\limits_{s=1}^{n-3}\left(\left(\prod\limits_{i=1}^s x_i-1\right)\left(x_{s+1}-1\right)\right). \end{align} $$

Calculating directly, we notice that the following equality holds true

(2.7) $$ \begin{align} \nonumber -a\prod\limits_{i=1}^{n-1} x_i+a\left(\prod\limits_{i=1}^{n-2} x_i-1\right)\left(x_{n-1}-1\right)=\\ -a\prod\limits_{i=1}^{n-1} x_i+a\prod\limits_{i=1}^{n-1} x_i-ax_{n-1}-a\prod\limits_{i=1}^{n-2} x_i +a=a-ax_{n-1}-a\prod\limits_{i=1}^{n-2} x_i. \end{align} $$

From equations (2.6) and (2.7), and then using the inductive assumption (2.3), we obtain

$$ \begin{align*} \left(a\prod\limits_{i=1}^{n-1} x_i-1\right)\left(ax_{n}-1\right)+a\sum\limits_{s=1}^{n-2}\left(\left(\prod\limits_{i=1}^s x_i-1\right)\left(x_{s+1}-1\right)\right)\\ =a^2\prod\limits_{i=1}^n x_i-ax_n+1+a-ax_{n-1}-a\prod\limits_{i=1}^{n-2} x_i+ a\sum\limits_{s=1}^{n-3}\left(\left(\prod\limits_{i=1}^s x_i-1\right)\left(x_{s+1}-1\right)\right)\displaystyle_{=}^{(2.3)} \\ a^2\prod\limits_{i=1}^n x_i-ax_n+1+a-ax_{n-1}-a\prod\limits_{i=1}^{n-2} x_i -\left(a\prod\limits_{i=1}^{n-2} x_i-1\right)\left(ax_{n-1}-1\right)+\\ +a^2\prod\limits_{i=1}^{n-1} x_i-a\sum\limits_{i=1}^{n-1} x_i+a(n-3)+1=a^2\prod\limits_{i=1}^n x_i-a\sum\limits_{i=1}^nx_i+a(n-2)+1. \end{align*} $$

Thus, assuming $T(n-1)$ , we have shown that $T(n)$ holds, completing the inductive step and concluding the proof of the lemma.

Theorem 2.2 Let $a,k\in \mathbb {N},\, b\in \mathbb {N}\cup \{0\}$ . For any integer $n\ge 2$ , the system of Diophantine equations

(2.8) $$ \begin{align} \left\{ \begin{array}{cll} x_{1,1}+x_{1,2}+\cdots+x_{1,n} & =&ax_{2,1}\cdot x_{2,2}\cdot \ldots \cdot x_{2,n}+b\, ,\\ x_{2,1}+x_{2,2}+\cdots+x_{2,n} & =&ax_{3,1}\cdot x_{3,2}\cdot \ldots \cdot x_{3,n}+b\, ,\\ &\ldots&\\ x_{k-1,1}+x_{k-1,2}+\cdots+x_{k-1,n}& =&ax_{k,1}\cdot x_{k,2}\cdot \ldots \cdot x_{k,n}+b\, ,\\ x_{k,1}+x_{k,2}+\cdots+x_{k,n}& =&ax_{1,1}\cdot x_{1,2}\cdot \ldots \cdot x_{1,n}+b \end{array} \right.\end{align} $$

has only finite number of solutions $x_{i,j}$ which are natural numbers.

Proof By adding sides of equations of the system of equations (2.8), we obtain

$$ \begin{align*} \sum\limits_{i=1}^k\sum\limits_{j=1}^n x_{i,j}=\sum\limits_{i=1}^ka\prod\limits_{j=1}^n x_{i,j}+kb. \end{align*} $$

Hence,

$$ \begin{align*} \sum\limits_{i=1}^k\left(a^2\prod\limits_{j=1}^n x_{i,j}-a\sum\limits_{j=1}^n x_{i,j}+a(n-2)+1\right)=k(a(n-2)+1)-kab. \end{align*} $$

By (2.1), we have

(2.9) $$ \begin{align} \nonumber \sum\limits_{i=1}^k \left(\left(a\prod\limits_{j=1}^{n-1}x_{i,j}-1\right) \left(ax_{i,n}-1\right)+a\sum\limits_{s=1}^{n-2}\left(\prod\limits_{j=1}^s x_{i,j}-1\right)\left(x_{i,s+1}-1\right)\right)=\\k(a(n-2)+1)-kab. \end{align} $$

For given $a,k,b,n$ , the number of solutions of equation (2.9) in positive integers is bounded above. Hence, the system of equations (2.8) has only a finite number of solutions in positive integers $x_{i,j}.$

Taking $k=1$ , an immediate consequence of Theorem 2.2 is the following result.

Corollary 2.3 For given $a\in \mathbb {N},\,b\in \mathbb {N}\cup \{0\}$ and any integer $n\ge 2$ , the number of solutions of the equation

(2.10) $$ \begin{align} x_{1}+x_{2}+\cdots+x_{n} =ax_{1}\cdot x_{2}\cdot \ldots \cdot x_{n}+b \end{align} $$

in positive integers $x_1\ge x_2\ge \cdots \ge x_n\ge 1$ is finite. In particular, in the case $b=0$ , the number of solutions $N_a(n)$ is finite.

Remark 2.4 Theorem 2.2 is true for all $a,b\in \mathbb {Q}, a\ge 1$ .

Remark 2.5 In the case of $b=0$ , we can provide a different proof of Corollary 2.3. Let $z_i = x_1x_2\cdot \ldots \cdot x_{i-1}x_{i+1}\cdot \ldots \cdot x_n = \frac {1}{x_i}\prod \limits _{j=1}^nx_j\in \mathbb {N}$ for $i\in \{1,2,\ldots ,n\}$ . Notice that from the inequality $x_1\ge x_2\ge \cdots \ge x_n\ge 1$ , we get the inequality $1 \le z_1\le z_2\le \cdots \le z_n$ . Then, equation (2.10) takes the form

(2.11) $$ \begin{align} \tfrac{1}{z_{1}}+\tfrac{1}{z_{2}}+\cdots+\tfrac{1}{z_{n}} =a\ge 1. \end{align} $$

Equation (2.11) has finitely many solutions in positive integers, as we can find upper bounds on $z_i$ . The bounds we will find are not optimal, but they are sufficient for our purposes. If $n\ge 2$ , then $ax_1 x_2\cdot \ldots \cdot x_n=x_1+x_2+\cdots +x_n\ge x_1+x_2\ge x_1+1>x_1$ , and hence $ax_2\cdot \ldots \cdot x_n\ge 2$ . From here, we can deduce

$$ \begin{align*}(n-1)x_2\ge x_2+\cdots+x_n=x_1(ax_2\cdot\ldots\cdot x_n-1)\ge x_1.\end{align*} $$

Therefore, $nx_2>x_1$ and $nz_1>z_2$ . We also have for $k\in \{2,3,\ldots ,n-1\}$ , that

$$ \begin{align*}nz_1 z_2\cdot \ldots\cdot z_k\ge z_1z_2\ge \prod\limits_{i=1}^nx_i\ge z_{k+1}.\end{align*} $$

Thus, for all $k\in \{1,2,\ldots ,n-1\}$ , we have $z_{k+1}\le nz_1\cdot z_2\cdot \ldots \cdot z_k$ . Now we can proceed with the inductive proof of the upper bound: $z_i\le a^{-1}n^{2^{i-1}},$ where $i\in \{1,2,\ldots ,n\}$ . Base step, as the $z_i$ are increasing, we can use equation (2.11) to obtain an inequality:

$$ \begin{align*}\tfrac{n}{z_1}\ge\tfrac{1}{z_{1}}+\tfrac{1}{z_{2}}+\cdots+\tfrac{1}{z_{n}} =a\ge 1,\,\mathrm{hence}\,z_1\le a^{-1}n.\end{align*} $$

If we now make the assumption that $z_i \leq a^{-1}n^{2^{i-1}}$ for all $i \in \{ 1, 2, \ldots , k\}$ , where $k < n$ , then $ z_{k+1}\le nz_1z_2\cdot \ldots \cdot z_k \leq n\frac {n^{2^0+2^1+2^2+\cdots +2^{k-1}}}{a} = \frac {n^{2^k}}{a}$ ; this establishes the inductive step.

The proof of Theorem 2.2 can be modified in the specific case of $a,n$ to create an efficient algorithm for finding solutions to equation (2.10).

Kurlandchik and Nowicki [Reference Kurlandchik and Nowicki6, Theorem 3] had earlier shown that $N_1(n)$ is finite for any $n\ge 2$ .

Schinzel’s question can be generalized. For given $a\in \mathbb {N}$ , does the number $N_a(n)$ tend to infinity with n? We can show with the elementary method the following theorems.

Theorem 2.6 If $a,n\in \mathbb {N}$ , then $\limsup \limits _{n\to \infty } N_a(n)=\infty .$

Proof We shall consider two cases. Let $a\in \{1,2\}$ . If $t\in \{0,1,\ldots ,\left \lfloor \frac {s}{2}\right \rfloor \}$ , where s is a nonnegative integer, then

$$ \begin{align*} \tfrac{1}{a}((a+1)^{s-t}+1)+\tfrac{1}{a}((a+1)^t+1)+\underbrace{1+1+\cdots+1}_{\tfrac{1}{a}((a+1)^s-1)\,\,\mathrm{times}}=\\ a\cdot \tfrac{1}{a}((a+1)^{s-t}+1)\cdot\tfrac{1}{a}((a+1)^t+1)\cdot \underbrace{1\cdot 1\cdot\ldots\cdot1}_{\tfrac{1}{a}((a+1)^s-1)\,\,\mathrm{times}}. \end{align*} $$

We have $s-t\ge t$ and $\tfrac {1}{a}((a+1)^{i}+1)\in \mathbb {N}$ , where i is a nonnegative integer. Hence, $N(\tfrac {1}{a}((a+1)^s+2a-1))\ge \left \lfloor \frac {s}{2}\right \rfloor +1.$ Therefore, $\limsup \limits _{n\to \infty } N_a(n)=\infty .$

Let $a\ge 3$ . If $t\in \{1,\ldots ,\left \lfloor \frac {s+1}{2}\right \rfloor \}$ , where $s\in \mathbb {N}$ , then

$$ \begin{align*} \tfrac{1}{a}((a-1)^{2s-2t+1}+1)+\tfrac{1}{a}((a-1)^{2t-1}+1)+\underbrace{1+1+\cdots+1}_{\tfrac{1}{a}((a-1)^{2s}-1)\,\,\mathrm{times}}=\\[-2pt] a\cdot \tfrac{1}{a}((a-1)^{2s-2t+1}+1)\cdot\tfrac{1}{a}((a-1)^{2t-1}+1)\cdot \underbrace{1\cdot 1\cdot\ldots\cdot1}_{\tfrac{1}{a}((a-1)^{2s}-1)\,\,\mathrm{times}}. \end{align*} $$

We have $2s-2t+1\ge 2t-1$ and $\tfrac {1}{a}((a-1)^{2i-1}+1), \tfrac {1}{a}((a-1)^{2i}-1) \in \mathbb {N}$ , where $i\in \mathbb {N}$ . Hence, $N(\tfrac {1}{a}((a-1)^{2s}+2a-1))\ge \left \lfloor \frac {s+1}{2}\right \rfloor .$

Therefore, $\limsup \limits _{n\to \infty } N_a(n)=\infty .$

Remark 2.7 Let $a\ge 3$ . Depending on the choice of $a\le n$ , equation (1.2) may not have solutions. The simplest example is $a=3$ and $n=4$ . In this case, equation (1.2) is equivalent to

$$ \begin{align*} (3x_1x_2x_3-1)(3x_4-1)+3(x_1x_2-1)(x_3-1)+3(x_1-1)(x_2-1)=7, \end{align*} $$

but the corresponding equation has no integer solutions $x_1\ge x_2\ge x_3\ge x_4\ge 1$ . This gives $N_3(4)=0.$

Remark 2.8 Due to the solutions , where $m\in \mathbb {N}$ and certain technical computations based on the method from Remark 2.5, we can prove that:

  1. (1) $N_a(a)=N_a(2a-1)=N_a(3a-2)=N_a(4a-3)=1$ , where $a\ge 2$ ,

  2. (2) $N_2(6)=2,\, N_a(4a-2)=1,$ where $a\ge 3$ ,

  3. (3) $N_a(n)=0$ if $n\in ((a,2a-1)\cup (2a-1,3a-2)\cup (3a-2,4a-3))\cap \mathbb {N},$

  4. (4) $N_a(ma-m+1)\ge 1$ , where $m\in \mathbb {N}$ .

Points (1)–(3) partially explain the basic structure of the right side of Table 1.

Table 1 The table shows values of $N_a(n)$ for small natural numbers $a\le n\le 10$ . The bold numbers are $N_a(n)$ , such that $n \ge 4a-1$ .

It has been proven in [Reference Zakarczemny15] that in the case of $a=1$ , the following theorem holds.

Theorem 2.9 If $n\in \mathbb {N},\,n\ge 2$ , then

(2.12) $$ \begin{align} N_1(n)\ge\left\lfloor\frac{d(n-1)+1}{2}\right\rfloor+\left\lfloor\frac{d(2n-1)+1}{2}\right\rfloor-1, \end{align} $$

where $d(j)$ is the number of positive divisors of $j.$ Moreover,

(2.13) $$ \begin{align} \nonumber N_1(n)\ge\left\lfloor\frac{d(n-1)+1}{2}\right\rfloor+\left\lfloor\frac{d(2n-1)+1}{2}\right\rfloor-1\\ +\left\lfloor\frac{d_2(3n+1)+1}{2}\right\rfloor+\left\lfloor\frac{d_3(4n+1)+1}{2}\right\rfloor+\left\lfloor\frac{d_3(4n+5)+1}{2}\right\rfloor\\ \nonumber-\delta(2|n+1)-\delta(3|n+1)-\delta(3|n+2)\\ \nonumber-\delta(5|n+2,\, n\ge 8)-\delta(7|n+3,\, n\ge 11)-\delta(11|n+4,\, n\ge 29), \end{align} $$

where $d_i(m)$ is the number of positive divisors of m which lie in the arithmetic progression ${i}\pmod {i+1}$ . The function $\delta $ is the Dirac delta function.

Remark 2.10 In the case $a=2$ , equation (1.2) has at least one typical solution in the form $(n-1,\underbrace {1,1,\ldots ,1}_{n-1\,\,\mathrm {times}}).$ Therefore, $N_2(n)\ge 1$ for all integers $n\ge 2.$

3 Main results

We give a lower bound on the number of solutions $N_a(n)$ of equation (1.2).

Theorem 3.1 If $a,n\in \mathbb {N},\,n\ge 2$ , then

(3.1) $$ \begin{align} N_a(n)\ge\left\lfloor\tfrac{d_{a-1}(a(n-2)+1)+1}{2}\right\rfloor+ \left\lfloor\tfrac{d_{2a-1}(2a(n-1)+1)+1}{2}\right\rfloor-\delta(2a-1|n), \end{align} $$

where $d_i(m)$ is the number of positive divisors of m which lie in the arithmetic progression $i \pmod {i+1}$ . The function $\delta $ is the Dirac delta function.

Proof In the set $\mathbb {N}^n$ , we have the following pairwise disjoint families of pairwise different $(x_1,x_2,\ldots ,x_n)$ solutions of equation (1.2). Note that in each case $x_i$ is an integer and $x_1\ge x_2\ge \cdots \ge x_n\ge 1$ . We define

$$ \begin{align*} A_1(n)=\{(\tfrac{n-2+\tfrac{d+1}{a}}{d},\tfrac{d+1}{a},\underbrace{1,1,\ldots,1}_{n-2\,\,\mathrm{times}}):\\ \,\,a(n-2)+1\equiv0\quad\pmod d,\,d\equiv-1\quad\pmod a,\,\\1\le d\le\sqrt{a(n-2)+1},\,d\in\mathbb{N}\}. \end{align*} $$

We also define

$$ \begin{align*} A_2(n)=\{(\tfrac{n-1+\tfrac{d+1}{2a}}{d},\tfrac{d+1}{2a},2,\underbrace{1,1,\ldots,1}_{n-3\,\,\mathrm{times}}):\\ \,\,2a(n-1)+1\equiv0\quad\pmod d,\,d\equiv -1\quad\pmod {2a},\,\\4a-1\le d\le\sqrt{2a(n-1)+1},\,d\in\mathbb{N}\}, \mathrm{\,\,when\,\,} n\ge 3. \end{align*} $$

We have $A_2(2)=\emptyset .$ Moreover,

$$ \begin{align*} |A_1(n)|= |\{d:\,\, a(n-2)+1\equiv0\quad\pmod d ,\,d\equiv-1\quad\pmod a,\,\\1\le d\le\sqrt{a(n-2)+1},\,d\in\mathbb{N} \}| =\left\lfloor\tfrac{d_{a-1}(a(n-2)+1)+1}{2}\right\rfloor. \end{align*} $$

In the case of the set $A_2(n)$ , we have $d\neq 2a-1$ ; thus,

$$ \begin{align*} |A_2(n)|=|\{d:\,\, 2a(n-1)+1\equiv0\quad\pmod d,\,d\equiv-1\quad\pmod {2a},\,\\ 4a-1\le d\le\sqrt{2a(n-1)+1},\,d\in\mathbb{N}\}|=\\=|\{d:\,\, 2a(n-1)+1\equiv0\quad\pmod d,\,d\equiv-1\quad\pmod {2a},\,\\ 1\le d\le\sqrt{2a(n-1)+1},\,d\in\mathbb{N}\}|\\-|\{d: 2a(n-1)+1\equiv0\quad\pmod d,\,d=2a-1\}|=\\ \left\lfloor\tfrac{d_{2a-1}(2a(n-1)+1)+1}{2}\right\rfloor-\delta(2a-1|n). \end{align*} $$

The sets $A_1(n),\,A_2(n)$ are disjoint. Hence, $N_a(n)\ge |A_1(n)|+|A_2(n)|.$ Thus, we get immediately (3.1).

Corollary 3.2 If $n\in \mathbb {N},\,n\ge 2$ , then

(3.2) $$ \begin{align} N_2(n)\ge\left\lfloor\tfrac{d(2n-3)+1}{2}\right\rfloor+ \left\lfloor\tfrac{d_{3}(4n-3)+1}{2}\right\rfloor-\delta(3|n). \end{align} $$

The following corollary is almost immediate.

Corollary 3.3 If $n\in \mathbb {N},\,n\ge 2$ , then

(3.3) $$ \begin{align} N_2(n)\ge \tfrac{1}{2}d(2n-3). \end{align} $$

Proof Formula (3.3) follows at once from Corollary 3.2 and inequalities

$$ \begin{align*}\left\lfloor\tfrac{d_{3}(4n-3)+1}{2}\right\rfloor\ge \delta(3|n),\,\,\left\lfloor\tfrac{x+1}{2}\right\rfloor\ge \tfrac{1}{2}x,\mathrm{\,\, where\,\,}x\in\mathbb{Z}.\\[-36pt]\end{align*} $$

For the convenience of the reader, values of $N_2(n)$ for small values of n are presented in Table 2.

Corollary 3.4 Let $n\in \mathbb {N},\,n\ge 3$ . If the equation

(3.4) $$ \begin{align} x_{1}+x_{2}+\cdots+x_{n} =2x_{1}\cdot x_{2}\cdot \ldots \cdot x_{n} \end{align} $$

has exactly one solution $(n-1,\underbrace {1,1,\ldots ,1}_{n-1\,\,\mathrm {times}})$ in the natural numbers $x_1\ge x_2\ge \cdots \ge x_n\ge 1$ , then $2n-3$ is a prime number.

Table 2 The table lists the numbers $N_2(n)$ for $2\le n\le 51$ .

Proof If $N_2(n)=1$ , then by Corollary 3.3 we get $2\ge d(2n-3)$ . Since $2n-3\ge 3$ , it follows that $2n-3$ is a prime number.

Remark 3.5 If $N_1(n)=1$ , then $n-1$ must be a Sophie Germain prime number (see [Reference Nyblom8]).

4 The set of exceptional values

Let $E_{\le k}^2=\{n:\,N_2(n)\le k,\,n\ge 2\}$ , where $k\in \mathbb {N}$ . In particular, $E_{\le 1}^2=\{n:\,N_2(n)=1, n\ge 2\}$ .

Theorem 4.1 The set $E_{\le k}^2$ has natural density $0$ , i.e., the ratio

$$ \begin{align*} \tfrac{1}{x}|E_{\le k}^2\cap [1,x]| \end{align*} $$

tends to $0$ as $x\to \infty $ .

Proof Let $\mathrm {\Omega }(m)$ count the total number of prime factors of m. We have $\mathrm {\Omega }(m)\le d(m)-1$ for every natural m. Let $\pi _i(x)=|\{m:\,\,\mathrm {\Omega }(m)=i,\,\,1\le m\le x\}|$ , i.e., the number of $1\le m\le x$ with i prime factors (not necessarily distinct). By Corollary 3.3, we have $N_2(n)\ge \frac {1}{2}d(2n-3).$ Thus, if $n\in E_{\le k}^2$ , then $d(2n-3)\le 2k$ and consequently $\mathrm {\Omega }(2n-3)\le 2k-1.$ Therefore,

$$ \begin{align*} |E_{\le k}^2\cap [1,x]|\le \sum\limits_{i=0}^{2k-1}\pi_i(2x-3), \end{align*} $$

where $x\ge 2.$ Using the sieve of Eratosthenes, one can show that (see [Reference Cojocaru and Murty2, p. 75])

$$ \begin{align*} \pi_i(x)\le \tfrac{1}{i!}x\tfrac{(A\log{\log{x}+B})^i}{\log{x}} \end{align*} $$

for some constants $A,B>0.$ There follows that

$$ \begin{align*} 0\le \tfrac{1}{x}|E_{\le k}^2\cap [1,x]|\le \tfrac{2x-3}{x}\sum\limits_{i=0}^{2k-1}\tfrac{1}{i!}\tfrac{(A\log{\log{(2x-3)}+B})^i}{\log{(2x-3)}}. \end{align*} $$

For a fixed k, the right-hand side tends to $0$ , as $x\to \infty $ . Thus,

$$ \begin{align*} \lim\limits_{x\to \infty} \tfrac{1}{x}|E_{\le k}^2\cap [1,x]|=0. \end{align*} $$

This completes the proof.

The above theorem implies that the set $E_k^2=\{n:\,\,N_2(n)=k,\,n\ge 2\}$ has zero natural density for any fixed $k\ge 1$ . This observation might suggest that the set ${E_k^2=\{n:\,\,N_2(n)=k,\,n\ge 2\}}$ is finite for any fixed $k\ge 1$ and that the number $N_2(n)\to \infty $ as $n\to \infty $ . In the next theorem, we study the average behavior of $N_2(n).$

Theorem 4.2 If $\epsilon>0$ , then for sufficiently large x, we have

$$ \begin{align*} \sum_{1<n\le x}N_2(n)\ge \tfrac{1-\epsilon}{8}x\log{x}. \end{align*} $$

Proof By [Reference Sándor, Mitrinović and Crstici9, Reference Tolev14], there exists constant $c>0$ such that

$$ \begin{align*}\left|\sum\limits_{\substack{1\le n\le x,\\n\equiv1\ \pmod 2}}d(n)-\tfrac{x}{4}\log{x}\right|\le cx,\end{align*} $$

for sufficiently large $x>x_0.$ It follows that

$$ \begin{align*}\sum\limits_{\substack{1\le n\le x,\\n\equiv1\ \pmod 2}}d(n) \ge \tfrac{x}{4}\log(x)-cx\end{align*} $$

for $x>x_0.$ By Corollary 3.3, for $n\ge 2$ , we have $N_2(n)\ge \frac {1}{2}d(2n-3)$ . Therefore,

$$ \begin{align*}\frac{1}{x}\sum\limits_{1< n\le x}N_2(n)\ge \frac{1}{x}\sum\limits_{1<n\le x}\tfrac{1}{2}d(2n-3)=\frac{1}{2x}\sum\limits_{\substack{1\le m\le 2x-3\\ m\equiv1\ \pmod 2}}d(m)\\ \ge \frac{1}{8}\log{(2x-3)}-c\tfrac{2x-3}{2x} \end{align*} $$

for $2x-3>x_0.$ Let $\epsilon>0$ , then for sufficiently large x, we have

$$ \begin{align*} \frac{1}{x}\sum\limits_{1< n\le x}N_2(n)\ge (1-\epsilon)\frac{1}{8}\log{x}.\\[-42pt] \end{align*} $$

References

Cassels, J. W. S., An introduction to Diophantine approximation, Chap. II, Cambridge University Press, Cambridge, 1957.Google Scholar
Cojocaru, A. C. and Murty, M. R., An introduction to sieve methods and their applications, Cambridge University Press, Cambridge, 2005.Google Scholar
Ecker, M. W., When does a sum of positive integers equal their product? Math. Mag. 75(2002), no. 1, 4147.Google Scholar
Guy, R. K., Unsolved problems in number theory (Section D24), Springer, New York–Heidelberg–Berlin, 2004.Google Scholar
Hurwitz, A., Über eine Aufgabe der unbestimmten analysis . Arch. Math. Phys. 3(1907), 185196.Google Scholar
Kurlandchik, L. and Nowicki, A., When the sum equals the product . Math. Gaz. 84(2000), no. 499, 9194.Google Scholar
Markoff, A. A., Sur les formes binaires indéfinies . Math. Ann. 17(1880), 379399.Google Scholar
Nyblom, M. A., Sophie Germain primes and the exceptional values of the equal-sum-and-product problem . Fibonacci Quart. 50(2012), no. 1, 5861.Google Scholar
Sándor, J., Mitrinović, D. S., and Crstici, B., Handbook of number theory I, Springer, Dordrecht, 2006.Google Scholar
Schinzel, A., Sur une propriété du nombre de diviseurs . Publ. Math. Debrecen 3(1954), 261262.Google Scholar
Schinzel, A., Selecta. Unsolved problems and unproved conjectures, Vol. 2, American Mathematical Society, Providence, RI, 2007, 1367.Google Scholar
Shiu, P., On Erdös’s last equation . Amer. Math. Mon. 126(2019), no. 9, 802808.Google Scholar
Takeda, W., On solutions to Erdős’ last equation . Integers 21(2021), A117.Google Scholar
Tolev, D. I., On the division problem in arithmetic progressions . C. R. Acad. Bulg. Sci. 41(1988), 3336.Google Scholar
Zakarczemny, M., On the equal sum and product problem . Acta Math. Univ. Comenian. 90(2021), no. 4, 387402.Google Scholar
Figure 0

Table 1 The table shows values of $N_a(n)$ for small natural numbers $a\le n\le 10$. The bold numbers are $N_a(n)$, such that $n \ge 4a-1$.

Figure 1

Table 2 The table lists the numbers $N_2(n)$ for $2\le n\le 51$.